Health Assessment Chapter 22 Prep-U Neurological Assessment, Neurological Assessment PrepU, Neurological Assessment (PrepU), Chapter 25: Assessing Neurologic System, Chapter 25: Assessing Neurologic System, Assessment Ch 25 Neuro System PrepU, PrepU...

अब Quizwiz के साथ अपने होमवर्क और परीक्षाओं को एस करें!

A 53-year-old man presents to the emergency department with a chief complaint of inability to form words, and numbness and weakness of the right arm and leg. Where would you locate the site of injury? 1- Left frontoparietal region 2- Right frontoparietal region 3- Left basal ganglia 4- Left temporal region

1

A 77-year-old female patient who is recovering in the hospital from a total knee replacement has rung her call bell and told the nurse that she needs pain medication. When assessing the patient's pain, what principle should the nurse bear in mind? 1- Older adults tend to have a blunted pain sensation, so complaints should be followed-up promptly. 2- Older adults frequently confuse pain with other tactile sensations. 3- Pain in older adults is often unrelated to physical harm or pathophysiological processes. 4- The sensation of pain increases with age, so older adults typically feel more pain for a longer period than younger patients.

1

A client presents to the emergency department status post-seizure. The physician wants to know what the pressure is in the client's head. What test might be ordered on this client? 1- Lumbar puncture 2- Echoencephalography 3- Nerve conduction studies 4- EMG

1

A nurse is caring for a client with lower back pain who is scheduled for myelography using metrizamide (a water-soluble contrast dye). After the test, the nurse should place the client in which position? 1- Head of the bed elevated 45 degrees 2- Prone 3- Supine with feet raised 4- Supine with the head lower than the trunk

1

A nurse is conducting a neurological assessment of a patient who has just been admitted to the unit. In preparation for assessing the patient for pronator drift, what instructions should the nurse provide to the patient? 1- "Please hold your arms straight out with your palms pointing up to the ceiling." 2- "Please close your eyes and then walk a few steps with one foot directly in front of the other." 3- "Please close your eyes and then touch the tip of your nose with one index finger and then the other." 4- "Please lift one leg a few inches off the bed and hold it as still as possible."

1

A nurse is performing a neurologic assessment on a client with a stroke and cannot elicit a gag reflex. This deficit is related to cranial nerve (CN) X, the vagus nerve. What will the nurse consider a priority nursing diagnosis? 1- Risk for aspiration 2- Risk for falls 3- Risk for impaired skin integrity 4- Decreased intracranial adaptive capacity

1

A nurse is performing a neurologic assessment on the client and notes a positive Romberg test. This test for balance is related to which of the following cranial nerves? 1- VIII 2- X 3- III 4- VII

1

A patient arrives to have an MRI done in the outpatient department. What information provided by the patient warrants further assessment to prevent complications related to the MRI? 1- "I am trying to quit smoking and have a patch on." 2- "I have been trying to get an appointment for so long." 3- "I have not had anything to eat or drink since 3 hours ago." 4- "My legs go numb sometimes when I sit too long."

1

A patient sustained a head injury during a fall and has changes in personality and affect. What part of the brain does the nurse recognize has been affected in this injury? 1- Frontal lobe 2- Parietal lobe 3- Occipital lobe 4- Temporal lobe

1

Cranial nerve IX is also known as which of the following? 1- Glossopharyngeal 2- Vagus 3- Spinal accessory 4- Hypoglossal

1

The critical care nurse is giving end-of-shift report on a client she is caring for. The nurse uses the Glasgow Coma Scale (GCS) to assess the level of consciousness (LOC) of a female client and reports to the oncoming nurse that the client has an LOC of 6. What does an LOC score of 6 in a client indicate? 1- Comatose 2- Somnolence 3- Stupor 4- Normal

1

The nurse has completed evaluating the client's cranial nerves. The nurse documents impairment of the right cervical nerves (CN IX and CN X). Based on these findings, the nurse should instruct the client to 1- refrain from eating or drinking for now. 2- have their spouse bring in the client's glasses. 3- wear any hearing aids while in the hospital. 4- use the walker when walking.

1

What part of the brain controls and coordinates muscle movement? 1- Cerebellum 2- Cerebrum 3- Midbrain 4- Brain stem

1

Which lobe of the brain is responsible for concentration and abstract thought? 1- Frontal 2- Parietal 3- Temporal 4- Occipital

1

Which lobe of the brain is responsible for spatial relationships? 1- Parietal 2- Temporal 3- Occipital 4- Frontal

1

Which of the following is a sympathetic nervous system effect? 1- Decreased peristalsis 2- Decreased blood pressure 3- Constricted pupils 4- Constricted bronchioles

1

Which of the following terms is used to describe rapid, jerky, involuntary, purposeless movements of the extremities? 1- Chorea 2- Bradykinesia 3- Spondylosis 4- Dyskinesia

1

Which term describes the fibrous connective tissues that cover the brain and spinal cord? 1- Meninges 2- Dura mater 3- Arachnoid mater 4- Pia mater

1

The nurse is educating a group of people newly diagnosed with migraine headaches. What information should the nurse include in the educational session? Select all that apply. 1- Keep a food diary. 2- Maintain a headache diary. 3- Sleep no more than 5 hours at a time. 4- Exercise in a dark room. 5- Use St. John's Wort.

1,2

A client is waiting in a triage area to learn the medical status of family members following a motor vehicle accident. The client is pacing, taking deep breaths, and handwringing. Considering the effects in the body systems, the nurse anticipates that the liver will: 1- cease function and shunt blood to the heart and lungs. 2- convert glycogen to glucose for immediate use. 3- produce a toxic byproduct in relation to stress. 4- maintain a basal rate of functioning.

2

A critical care nurse is documenting her assessment of a client she is caring for. The client is status post-resection of a brain tumor. The nurse documents that the client is flaccid on the left. What does this mean? 1- The client has an abnormal posture response to stimuli. 2- The client is not responding to stimuli. 3- The client is hyperresponsive on the left. 4- The client is hyporesponsive on the left.

2

A high school soccer player sustained five concussions before she was told that she should never play contact sports again. After her last injury, she began experiencing episodes of double vision. She was told that she had most likely incurred damage to which cranial nerve? 1- V (Trigeminal) 2- VI (Abducens) 3- VII (Facial) 4- IV (Trochlear)

2

A middle-aged woman has scheduled an appointment with her nurse practitioner because she has been experiencing intractable muscle weakness in recent weeks. Which of the following characteristics of the patient's weakness should cause the nurse to suspect a neurological etiology? 1- The patient's weakness is most severe in the early morning. 2- The weakness is primarily on the left side of the patient's body. 3- The weakness is not relieved by increasing her food intake. 4- The patient's weakness began around the time of her husband's death.

2

A nurse is caring for a client with deteriorating neurologic status. The nurse is performing an assessment at the beginning of the shift that reveals a falling blood pressure and heart rate, and the client makes no motor response to stimuli. Which documentation of neuromuscular status is most appropriate? 1- Abnormal posture 2- Flaccidity 3- Weak muscular tone 4- Decorticate posturing

2

A nurse is working in a neurologist's office. The physician orders a Romberg test. The nurse should have the client: 1- touch his or her nose with one finger. 2- close his or her eyes and stand erect. 3- close his or her eyes and discriminate between dull and sharp. 4- close his or her eyes and jump on one foot.

2

A nursing educator is talking with nurses about the effects of the aging process and neurologic changes. What would the educator identify as a normal neurological change that accompanies the aging process? 1- Hyperactive deep tendon reflexes 2- Reduction in cerebral blood flow (CBF) 3- Increased cerebral metabolism 4- Hypersensitivity to painful stimuli

2

A patient has been brought to the emergency department (ED) with signs and symptoms of a stroke and a stat computed tomography (CT) head scan has been ordered. The ED nurse should know that the image that results from CT indicates distinguishing differences based on which of the following variables? 1- Proximity to the CT scanner 2- Variations in tissue density 3- Metabolic activity 4- Oxygen consumption

2

A patient with a neurological disorder is being assessed by the nurse. The nurse assesses the patient's biceps reflex as diminished. The nurse would be correct in documenting this response as what? 1- 0 2- 1+ 3- 2+ 4- 3+

2

Lower motor neuron lesions cause 1- increased muscle tone. 2- flaccid muscles. 3- no muscle atrophy. 4- hyperactive and abnormal reflexes.

2

The Glasgow Coma Scale is a common screening tool used for patients with a head injury. During the physical exam, the nurse documents that the patient is able to spontaneously open her eyes, obey verbal commands, and is oriented. The nurse records the highest score of: 1- 20 2- 15 3- 10 4- 5

2

The nurse is assessing the pupils of a patient who has had a head injury. What does the nurse recognize as a parasympathetic effect? 1- Dilated pupils 2- Constricted pupils 3- One pupil is dilated and the opposite pupil is normal 4- Roth's spots

2

The nurse is caring for a patient who was involved in a motor vehicle accident and sustained a head injury. When assessing deep tendon reflexes (DTR), the nurse observes diminished or hypoactive reflexes. How will the nurse document this finding? 1- 0 2- 1+ 3- 2+ 4- 3+

2

The nurse is doing an initial assessment on a patient recently admitted to the unit with a diagnosis of cerebrovascular accident (CVA). The patient has difficulty copying a figure that the nurse has drawn. The nurse uses this technique to assess for what type of aphasia? 1- Auditory-receptive 2- Visual-receptive 3- Expressive speaking 4- Expressive writing

2

The nurse is performing a detailed mental status assessment of an older adult patient who has a diagnosis of mild Alzheimer's disease. What assessment most accurately gauges the patient's abstract reasoning? 1- "What city and state are we in right now?" 2- "What would you do if you found a stamped envelope on the street?" 3- "If you divide 16 by four and then double it, what do you get?" 4- "How do you believe that Alzheimer's disease is affecting you?"

2

The nurse is performing a neurologic assessment on a client diagnosed with a stroke and cannot elicit a gag reflex. This deficit is related to which of the following cranial nerves? 1- VIII 2- X 3- III 4- VII

2

The nurse obtains a Snellen eye chart when assessing cranial nerve function. Which cranial nerve is the nurse testing when using the chart? 1- CN I 2- CN II 3- CN III 4- CN IV

2

There are 12 pairs of cranial nerves. Only three are sensory. Select the cranial nerve that is affected with decreased visual fields. 1- Cranial nerve I 2- Cranial nerve II 3- Cranial nerve III 4- Cranial nerve IV

2

Which cerebral lobes is the largest and controls abstract thought? 1- Temporal 2- Frontal 3- Parietal 4- Occipital

2

Which term refers to the inability to coordinate muscle movements, resulting difficulty walking? 1- Agnosia 2- Ataxia 3- Spasticity 4- Rigidity

2

Which term refers to the inability to coordinate muscle movements, resulting in difficulty walking? 1- Agnosia 2- Ataxia 3- Spasticity 4- Rigidity

2

Age-related changes in the neurologic system must be carefully assessed. Which of the following changes does the nurse expect to find in some degree depending on the patient's age and medical condition? Select all that apply. 1- Hyper-reactive deep tendon reflexes 2- Decreased muscle mass 3- Increased sensitivity to heat and cold 4- Stage IV sleep is prolonged 5- Increased sensitivity of taste buds 6- Reduced papillary responses

2,3,6

A 30-year-old primiparous woman has been admitted in early labor. The obstetrical nurse has read on the patient's prenatal record that she has a history of seizures. The nurse should understand that seizures most often occur as a result of: 1- Benign spinal cord lesions 2- Cranial nerve deficits 3- Abnormal activity in the cerebral cortex 4- Faulty integration of sensory impulses by the thalamus

3

A client in the emergency department has a suspected neurologic disorder. To assess gait, the nurse asks the client to take a few steps; with each step, the client's feet make a half circle. To document the client's gait, the nurse should use which term? 1- Ataxic 2- Dystrophic 3- Helicopod 4- Steppage

3

A client preparing to undergo a lumbar puncture states he doesn't think he will be able to get comfortable with his knees drawn up to his abdomen and his chin touching his chest. He asks if he can lie on his left side. Which statement is the best response by the nurse? 1- "Lying on your left side will be fine during the procedure." 2- "There's no other option but to assume the knee-chest position." 3- "Although the required position may not be comfortable, it will make the procedure safer and easier to perform." 4- "I'll report your concerns to the physician."

3

A client who was found unconscious at home is brought to the hospital by a rescue squad. In the intensive care unit, the nurse checks the client's oculocephalic (doll's eye) response by: 1- introducing ice water into the external auditory canal. 2- touching the cornea with a wisp of cotton. 3- turning the client's head suddenly while holding the eyelids open. 4- shining a bright light into the pupil.

3

A nurse has assessed a patient's orientation during the initial head-to-toe assessment near the beginning of a shift. The patient is able to state his own full name and knows the name of the hospital but is unable to identify the month and the year. How should the nurse best document this assessment finding? 1- "Patient is demonstrating signs of decreased neurological function." 2- "Patient is disoriented." 3- "Patient is oriented to person and place, but unable to state month and year." 4- "Patient is oriented to person, oriented to place, but not oriented to time."

3

A nurse is assessing reflexes in a patient with hyperactive reflexes. When the patient's foot is abruptly dorsiflexed, it continues to "beat" two to three times before setting into a resting position. How would the nurse document this finding? 1- Rigidity 2- Flaccidity 3- Clonus 4- Ataxia

3

A nurse is assisting during a lumbar puncture. How should the nurse position the client for this procedure? 1- Prone, with the head turned to the right 2- Supine, with the knees raised toward the chest 3- Lateral recumbent, with chin resting on flexed knees 4- Lateral, with right leg flexed

3

A nurse is providing education about migraine headaches to a community group. The cause of migraines has not been clearly demonstrated, but is related to vascular disturbances. A member of the group asks about familial tendencies. The nurse's correct reply will be which of the following? 1- "There is a very weak familial tendency." 2- "No familial tendency has been demonstrated." 3- "There is a strong familial tendency." 4- "Only secondary migraine headaches show a familial tendency."

3

A nurse is working on a neurological unit with a nursing student who asks the difference between primary and secondary headaches. The nurse's correct response will include which of the following statements? 1- "A secondary headache is one for which no organic cause can be identified." 2- "A secondary headache is located in the frontal area." 3- "A secondary headache is associated with an organic cause, such as a brain tumor." 4- "A migraine headache is an example of a secondary headache."

3

A patient comes to the emergency department with severe pain in the face that was stimulated by brushing the teeth. What cranial nerve does the nurse understand can cause this type of pain? 1- III 2- IV 3- V 4- VI

3

A patient has been diagnosed with a deficiency of the major neurotransmitter acetylcholine. Based on this information, the nurse knows to assess the patient for complications associated with: 1- Fine movements. 2- Sleep patterns. 3- Heart rate and rhythm. 4- Emotional balance.

3

A primary nursing assessment for a patient who has sustained a fracture involving the basilar skull is inspection for: 1- Leakage of CSF from the nose. 2- Ecchymosis of the mastoid process of the temporal bone. 3- Leakage of CSF from the ear. 4- Vomiting and headaches due to increased intracranial pressure.

3

The nurse receives a physician's order to administer 1,000 mL of intravenous (IV) normal saline solution over 8 hours to a client who recently had a stroke. What should the drip rate be if the drop factor of the tubing is 15 gtt/mL? Record your answer using a whole number.

31

A client is scheduled for an EEG. The client asks about any diet-related prerequisites before the EEG. Which diet-related advice should the nurse provide to the client? 1- Avoid eating food at least 8 hours before the test. 2- Include an increased amount of minerals in the diet. 3- Decrease the amount of minerals in the diet. 4- Avoid taking sedative drugs or drinks that contain caffeine for at least 8 hours before the test.

4

A client is weak and drowsy after a lumbar puncture. The nurse caring for the client knows that what priority nursing intervention should be provided after a lumbar puncture? 1- Administer antihistamines to the client. 2- Provide adequate caffeine-rich drinks to the client. 3- Assess the level of consciousness (LOC) and the pupil response of the client. 4- Position the client flat for at least 3 hours.

4

A nurse conducts the Romberg test by asking the client to stand with the feet close together and the eyes closed. As a result of this posture, the client suddenly sways to one side and is about to fall when the nurse intervenes and prevents the client from being injured. In which way should the nurse interpret the client's result? 1- Positive Romberg test, indicating a problem with level of consciousness 2- Negative Romberg test, indicating a problem with body mass 3- Negative Romberg test, indicating a problem with vision 4- Positive Romberg test, indicating a problem with equilibrium

4

A nurse is caring for a client with an injury to the central nervous system. When caring for a client with a spinal cord insult slowing transmission of the motor neurons, the nurse would anticipate a delayed reaction in: 1- identification of information due to slowed passages of information to brain. 2- cognitive ability to understand relayed information. 3- processing information transferred from the environment. 4- response due to interrupted impulses from the central nervous system

4

A nurse is completing a neurological assessment and determines that the client has significant visual deficits. Considering the functions of the lobes of the brain, which area will most likely contain the neurologic deficit? 1- frontal 2- parietal 3- temporal 4- occipital

4

A nurse is performing a neurologic assessment on a client. The nurse observes the client's tongue for symmetry, tremors, and strength, and assesses the client's speech. Which cranial nerve is the nurse assessing? 1- IV 2- IX 3- VI 4- XII

4

A patient is being tested for a gag reflex. When the nurse places the tongue blade to the back of the throat, there is no response elicited. What dysfunction does the nurse determine the patient has? 1- Dysfunction of the spinal accessory nerve 2- Dysfunction of the acoustic nerve 3- Dysfunction of the facial nerve 4- Dysfunction of the vagus nerve

4

A patient who has been diagnosed with colon cancer is scheduled to undergo positron emission tomography (PET) to search for metastases. In preparation for this diagnostic procedure, what teaching point should the nurse provide to the patient? 1- "Sound waves will be refracted throughout your body and a computer will analyze them." 2- "A series of X-rays will be taken that will be combined to create a three-dimensional image your body." 3- "It's very important that all metal objects be removed from your body before the test." 4- "You'll be given a radioactive substance that will be measured during the test."

4

A patient who has suffered a stroke is unable to maintain respiration and is intubated and placed on mechanical ventilator support. What portion of the brain is most likely responsible for the inability to breathe? 1- Frontal lobe 2- Occipital lobe 3- Parietal lobe 4- Brain stem

4

The nurse is admitting a patient to the unit who is diagnosed with a lower motor neuron lesion. What clinical manifestations would this patient most likely exhibit? 1- Increased muscle tone 2- No muscle atrophy 3- Hyperactive and abnormal reflexes 4- Absent or decreased reflexes

4

The nurse is assessing the throat of a client with throat pain. In asking the client to stick out the tongue, the nurse is also assessing which cranial nerve? 1- Cranial nerve I 2- Cranial nerve V 3- Cranial nerve XI 4- Cranial nerve XII

4

The physician's office nurse is caring for a client who has a history of a cerebral aneurysm. Which diagnostic test does the nurse anticipate to monitor the status of the aneurysm? 1- Myelogram 2- Electroencephalogram 3- Echoencephalography 4- Cerebral angiography

4

The pre-nursing class is learning about the nervous system in their anatomy class. What part of the nervous system would the students learn is responsible for digesting food and eliminating body waste? 1- Central 2- Sympathetic 3- Peripheral 4- Parasympathetic

4

The trochlear nerve controls which function? 1- Movement of the tongue 2- Hearing and equilibrium 3- Visual acuity 4- Eye muscle movement

4

To assess a client's cranial nerve function, a nurse should assess: 1- hand grip. 2- orientation to person, time, and place. 3- arm drifting. 4- gag reflex.

4

When learning about the nervous system, students learn that which nervous system regulates the expenditure of energy? 1- Parasympathetic 2- Central 3- Peripheral 4- Sympathetic

4

Which neurotransmitter demonstrates inhibitory action, helps control mood and sleep, and inhibits pain pathways? 1- Enkephalin 2- Norepinephrine 3- Acetylcholine 4- Serotonin

4

Which safety action will the nurse implement for a client receiving oxygen therapy who is undergoing magnetic resonance imaging (MRI)? 1- Securely fasten the client's portable oxygen tank to the bottom of the MRI table after the client has been positioned on the table. 2- Check the client's oxygen saturation level using a pulse oximeter after the client has been placed on the MRI table. 3- Note that no special safety actions need to be taken. 4- Ensure that no client care equipment containing metal enters the room where the MRI table is located.

4

nurse is assessing a client with a brain injury. What is a client's cerebral perfusion pressure (CPP) when the blood pressure (BP) is 90/50 mm Hg and the intracranial pressure (ICP) is 21? Round to the nearest whole number.

42

When evaluating a client's risk for cerebrovascular accident, which client would the nurse identify as being at highest risk?

68-year-old African American with hypertension

A client is scheduled for an EEG after having a seizure for the first time. Client preparation for this test should include which instruction? a) "Avoid stimulants and alcohol for 24 to 48 hours before the test." b) "Don't shampoo your hair for 24 hours before the test." c) "Don't eat anything for 12 hours before the test." d) "Avoid thinking about personal matters for 12 hours before the test."

A

A client recently experienced a stroke with accompanying left-sided paralysis. His family voices concerns about how to best interact with him. They report the client doesn't seem aware of their presence when they approach him on his left side. What advice should the nurse give the family? a) "The client is unaware of his left side. You should approach him on the right side." b) "The client is feeling an emotional loss. He'll eventually start acknowledging you on his left side." c) "This condition is temporary." d) "The client is unaware of his left side. You need to encourage him to interact from this side."

A

A client with a suspected brain tumor is scheduled for a computed tomography (CT) scan. What should the nurse do when preparing the client for this test? a) Determine whether the client is allergic to iodine, contrast dyes, or shellfish. b) Immobilize the neck before the client is moved onto a stretcher. c) Administer a sedative as ordered. d) Place a cap over the client's head.

A

A home health nurse visits a client who's taking pilocarpine, a miotic agent, to treat glaucoma. The nurse notes that the client's pilocarpine solution is cloudy. What should the nurse do first? a) Advise the client to discard the drug because it may have undergone chemical changes or become contaminated. b) Advise the client to keep the container closed tightly and protected from light. c) Watch the client or a family member administer the drug to determine possible contamination sources. d) Advise the client to obtain a fresh container of pilocarpine solution to avoid omitting ordered doses.

A

A nurse is caring for a client with a brain tumor and increased intracranial pressure (ICP). Which intervention should the nurse include in the care plan to reduce ICP? a) Administer stool softeners. b) Position the client with the head turned toward the side of the brain tumor. c) Provide sensory stimulation. d) Encourage coughing and deep breathing.

A

A parent of a child with a moderate head injury asks the nurse, "How will you know if my child is getting worse?" The nurse should tell the parents that best indicator of the child's brain function is: a) level of consciousness. b) reactions of the pupils. c) the vital signs. d) motor strength.

A

After 1 month of therapy, the client in spinal shock begins to experience muscle spasms in the legs, and calls the nurse in excitement to report the leg movement. Which response by the nurse would be the most accurate? a) "The movements occur from muscle reflexes that cannot be initiated or controlled by the brain." b) "These movements indicate that the damaged nerves are healing." c) "This is a good sign. Keep trying to move all the affected muscles." d) "The return of movement means that eventually you should be able to walk again."

A

Because of symptoms experienced after a cerebrovascular accident (CVA), the nurse discovers that a client needs assistance using utensils while eating. What would the nurse do to support this activity of care? a) Encourage participation in the feeding process to the best of the client's abilities. b) Feed another patient and wait in the dining room until the client can accomplish feeding. c) Request that the client's food be pureed by dietary staff. d) Have the family feed the client at every meal to reduce staffing limitations.

A

The client with retinal detachment in the right eye is extremely apprehensive and tells the nurse, "I am afraid of going blind. It would be so hard to live that way." What factor should the nurse consider before responding to this statement? a) Optimism is justified because surgical treatment has a 90% to 95% success rate. b) More and more services are available to help newly blind people adapt to daily living. c) Repeat surgery is impossible, so if this procedure fails, vision loss is inevitable. d) The surgery will only delay blindness in the right eye, but vision is preserved in the left eye.

A

The nurse is assessing a client with a head injury. On admission, the pupils were equal; now the right pupil is fully dilated and nonreactive, and the left pupil is 4 mm and reacts to light. What would this change in neurologic status of the client suggest to the nurse? a) Increased intracranial pressure b) Decreased intracranial pressure c) This is a normal response after a head injury, and the pupils will be expected to return to normal. d) The test was not performed accurately; there was too much light in the examination room.

A

The nurse is teaching a client with multiple sclerosis about prevention of urinary tract infection (UTI) and renal calculi. Which of the following nutrition recommendations by the nurse would be the most likely to reduce the risk of these conditions? a) Increase fluids (2500 mL/day) and maintain urine acidity by drinking cranberry juice. b) Eat foods containing vitamins C, D, and E, and drink at least 2 L of fluid a day. c) Drink a large amount of fluids, especially milk products, and eat a diet that includes multiple sources of vitamin D. d) Eat foods and ingest fluids that will cause the urine to be less acidic.

A

Which of the following clients requires increased sensory stimulation to prevent sensory deprivation? a) A 65-year-old client who has employment-induced presbycusis and advanced glaucoma b) A 24-year-old client who has been admitted with an anxiety disorder and appears very agitated c) An 84-year-old client who has hemiparesis and ambulates with a walker d) A 60-year-old client who is blind, reads books through use of Braille, listens to the radio, and regularly takes walks around the unit

A

Which of the following nursing intervention can prevent a client from experiencing autonomic dysreflexia? a) Monitoring the patency of an indwelling urinary catheter b) Assessing laboratory test results as ordered c) Placing the client in the Trendelenburg's position d) Administering zolpidem tartrate

A

The nurse is caring for a client with a diagnosis of cerebrovascular accident (CVA) with left-sided hemiparesis. What would be important nursing measures in the acute phase of care? Select all that apply. a) Turn and position every 2 hours. b) Perform passive range of motion on the affected side. c) Support the affected side with pillows. d) Perform active range of motion on both sides. e) Perform passive range of motion on both sides.

A,B,C

The nurse, caring for a client with Ménière's disease, needs to assist with what when the client is experiencing an attack? a) Sleeping b) ADLs c) Coughing d) URIs

ADLs

Cranial Nerve VI

Abducens: Motor controls lateral eye movements

What should the nurse assess to test the function of the occipital lobe?

Ability to read

A client with quadriplegia is in spinal shock. What finding should the nurse expect? a) Positive Babinski's reflex along with spastic extremities b) Spasticity of all four extremities c) Absence of reflexes along with flaccid extremities d) Hyperreflexia along with spastic extremities

Absence of reflexes along with flaccid extremities

The nurse is completing a neurologic assessment and uses the whisper test to assess which of the following cranial nerves?

Acoustic Clinical examination of the acoustic nerve can be done by the whisper test. Having the patient say "ah" tests the vagus nerve. Observing for symmetry when the patient performs facial movements tests the facial nerve. The olfactory nerve is tested by having the patient identify specific odors.

Cranial Nerve VIII

Acoustic, vestibulocochlear: Sesnroy contains sensory fibers for hearing and balance

A nurse assesses the patient's level of consciousness using the Glasgow Coma Scale. What score indicates severe impairment of neurologic function? a) 3 b) 15 c) 9 d) 6

Answer: 3

A client with cerebral metastasis suddenly experiences a seizure for which phenytoin 10 mg/kg intravenously is ordered as an initial loading dose. The client weighs 165 pounds. How many milligrams of phenytoin should the client receive? Enter the number ONLY.

Answer: 750

A client suffers a head injury. The nurse implements an assessment plan to monitor for potential subdural hematoma development. Which manifestation does the nurse anticipate seeing first? a) Alteration in level of consciousness (LOC) b) Decreased heart rate c) Bradycardia d) Slurred speech

Answer: A-alteration in LOC

A patient in the ER has bruising over the mastoid bone and rhinorrhea. The triage nurse suspects the patient has which type of skull fracture? a) Basilar b) Simple c) Comminuted d) Linear

Answer: A-basilar

A nurse is providing care to a client with Parkinson's disease. The nurse understands the the client's signs and symptoms are related to a depletion of which of the following? a) Dopamine b) Norepinephrine c) Serotonin d) Acetylcholine

Answer: A-dopamine

If untreated, squamous cell carcinoma of the external ear can spread through the temporal bone causing what effect? a) Facial nerve paralysis b) Diplopia c) Motor impairment d) Nystagmus

Answer: A-facial nerve paralysis

Mildred McCarthy, an 89-year-old retired government clerk, is being admitted to your rehabilitation hospital as a result of the tetraplegia caused by stroke. Her condition is stable, and after admission, she will begin physical and psychological therapy. An important part of your nursing management is to reposition Mrs. McCarthy every 2 hours. What is the rationale behind this intervention? a) Maintain sufficient integument capillary pressure b) Passive exercise c) Change of scenery d) Maintain psychological well-being

Answer: A-maintain sufficient integument capillary pressure

An aging client is brought to the eye clinic by the son. The son states he has seen his parent holding reading materials at an increasing distance to focus properly. What age-related changes does this indicate? a) Presbyopia b) Myopia c) Cataract d) Macular degeneration

Answer: A-presbyopia

Which nursing diagnosis takes highest priority for a client admitted for evaluation for Ménière's disease? a) Risk for injury related to vertigo b) Risk for deficient fluid volume related to vomiting c) Acute pain related to vertigo d) Imbalanced nutrition: Less than body requirements related to nausea and vomiting

Answer: A-risk for injury r/t vertigo

A client has a herniated disk in the region of the third and fourth lumbar vertebrae. Which nursing assessment finding most supports this diagnosis? a) Severe lower back pain b) Weakness and atrophy of the arm muscles c) Hypoactive bowel sounds d) Sensory deficits in one arm

Answer: A-severe lower back pain

Which of the following conditions occurs when there is bleeding between the dura mater and arachnoid membrane? a) Subdural hematoma b) Intracerebral hemorrhage c) Extradural hematoma d) Epidural hematoma

Answer: A-subdural hematoma

A client has sustained a traumatic brain injury with involvement of the hypothalamus. The nurse is concerned about the development of diabetes insipidus. Which of the following would be an appropriate nursing intervention to monitor for early signs of diabetes insipidus? a) Take daily weights. b) Assess for pupillary response frequently. c) Assess vital signs frequently. d) Reposition the client frequently.

Answer: A-take daily weights

A client is scheduled for a laminectomy to repair a herniated intervertebral disk. When developing the postoperative care plan, the nurse should include which action? a) Turning the client from side to side, using the logroll technique b) Keeping a pillow under the client's knees at all times c) Maintaining bed rest for 72 hours after the laminectomy d) Placing the client in semi-Fowler's position

Answer: A-turning the client from side to side, using the legroom technique

Which of the following surgical procedures involves flattening the anterior curvature of the cornea by removing a stromal lamella layer? a) Keratoplasty b) LASIK c) PRK d) Keratoconus

Answer: B- LASIK

A 17-year-old adolescent with a history of muscular dystrophy is admitted with aspiration pneumonia. The nurse asks the parents if the client has an advance directive. Which response by the parents leads the nurse to believe that the parents don't understand the severity of the client's medical condition? a) "He has pneumonia; I shouldn't have let him go to that party last week." b) "He is only 17. He doesn't need an advance directive." c) "This is the third time he's had pneumonia in the past 6 months. I'm afraid he needs a feeding tube." d) "Yes, he has an advance directive."

Answer: B-"He is only 17. He doesn't need an advance directive."

A patient is newly diagnosed with relapsing-remitting multiple sclerosis (RRMS). Which instruction should the nurse provide? a) "Your type of MS is the least common, making it difficult to manage." b) "You must avoid stress and extreme fatigue, because these can trigger a relapse." c) "You should take your medications only during times of relapse." d) "You will have a steady and gradual decline in function."

Answer: B-"You must avoid stress and extreme fatigue, because these can trigger a relapse."

A patient's vision is assessed at 20/200. The patient asks what that means. Which of the following is the most appropriate response by the nurse? a) "You see an object from 200 feet away that a person with normal vision sees from 20 feet away." b) "You see an object from 20 feet away that a person with normal vision sees from 200 feet away." c) "You see an object from 200 feet away that a person with normal vision sees from 200 feet away." d) "You see an object from 20 feet away that a person with normal vision sees from 20 feet away."

Answer: B-"You see an object from 20 feet away that a person with normal vision sees from 200 feet away."

Thrombolytic therapy should be initiated within what time frame of an ischemic stroke for best functional outcome? a) 12 hours b) 3 hours c) 9 hours d) 6 hours

Answer: B-3 hours

A patient has been diagnosed as having global aphasia. The nurse recognizes that the patient will be unable to do which of the following actions? a) Form understandable words b) Form understandable words and comprehend the spoken word c) Comprehend the spoken word d) Speak at all

Answer: B-Form understandable words and comprehend the spoken word

A nurse in a primary care office is getting a client ready for an examination with a health care provider. While talking to the client, she notices that her left upper eyelid is drooping. She records on the client's record that she observed which of the following? a) Ptolemy b) Ptosis c) Proptosis d) Nystagmus

Answer: B-Ptosis(dropping or falling of the lower lid)

A female patient with meningitis has a history of seizures. Which of the following activities should the nurse do while the patient is actively seizing? a) Provide oxygen or anticonvulsants, whichever is available b) Turn the patient to the side during a seizure and do not restrain movements c) Place a cooling blanket beneath the patient d) Suction the patient's mouth and pharynx

Answer: B-Turn the patient to the side during a seizure and do not restrain movements

Christin Victor, a 24-year-old blind medical transcriptionist, is a client in the hospital where you practice nursing and is awaiting an appendectomy. Which of the following steps would you take, as her nurse, to effectively communicate with Ms. Victor? a) Notify Ms. Victor before touching her. b) All options are correct. c) Let Ms. Victor know when you leave her room. d) Announce your entrance into Ms. Victor's room.

Answer: B-all options are correct

While snowboarding, a 17-year-old client fell and struck his head, resulting in a loss of consciousness. Within an hour after his arrival via squad at the ED where you practice nursing, he regained consciousness. He was admitted for 24-hour observation and was discharged without neurologic impairment. What would you expect the neurologist's diagnosis to be? a) Laceration b) Concussion c) Contusion d) Skull fracture

Answer: B-concussion

The primary North American vector transmitting arthropod-borne virus encephalitis is which of the following? a) Flea b) Mosquito c) Horse d) Tick

Answer: B-mosquito

A patient with herpes simplex virus (HSV) encephalitis is receiving acyclovir (Zovirax). To ensure early intervention, the nurse monitors laboratory values and urine output for which of the following adverse reactions? a) Integumentary b) Renal c) Hepatic d) Musculoskeletal

Answer: B-renal

Which of the following is the treatment of choice for acoustic neuromas? a) Radiation b) Surgery c) Chemotherapy d) Palliation

Answer: B-surgery

You are caring for a client with chronic migraines who is prescribed medication. What drug-related instructions should the nurse give the client? a) Take medication only when migraine is intense. b) Take medication as soon as symptoms of the migraine begin. c) Take medication just before going to bed at night. d) Take medication only during the morning when it's calm and quiet.

Answer: B-take meds as soon as symptoms of migraine begins

A client who has just been diagnosed with mixed muscular dystrophy asks the nurse about the usual course of this disease. How should the nurse respond? a) "You should ask your physician about that." b) "This form of muscular dystrophy is a relatively benign disease that progresses slowly." c) "You may experience progressive deterioration in all voluntary muscles." d) "The strength of your arms and pelvic muscles will decrease gradually, but this should cause only slight disability."

Answer: C-"You may experience progressive deterioration in all voluntary muscles."

A nurse is caring for a client with a brain tumor and increased intracranial pressure (ICP). Which intervention should the nurse include in the care plan to reduce ICP? a) Position the client with the head turned toward the side of the brain tumor. b) Provide sensory stimulation. c) Administer stool softeners. d) Encourage coughing and deep breathing.

Answer: C-administer stool softeners

The nurse is taking care of a patient with a history of headaches. The nurse takes measures to reduce headaches in the patient in addition to administering medications. Which of the following appropriate nursing interventions may be provided by the nurse to such a patient? a) Perform the Heimlich maneuver b) Use pressure-relieving pads or a similar type of mattress c) Apply warm or cool cloths to the forehead or back of the neck d) Maintain hydration by drinking eight glasses of fluid a day

Answer: C-apply cool or warn cloths to the forehead or back of the neck

The nurse is instructing a client's family members on the most incapacitating symptom of Ménière's disease. Which nursing instruction associated with the symptom is most helpful? a) Keep a bucket beside the bed. b) Sit in front of the client when speaking. c) Assist the client when ambulating. d) Ensure low lighting in the room.

Answer: C-assist the client w/ambulating

Which of the following is the earliest sign of increasing ICP? a) Posturing b) Vomiting c) Change in level of consciousness (LOC) d) Headache

Answer: C-change in LOC

When performing a postoperative assessment on a client who has undergone surgery to manage increased intracranial pressure (ICP), a nurse notes an ICP reading of 0 mm Hg. Which action should the nurse perform first? a) Continue the assessment because no actions are indicated at this time. b) Document the reading because it reflects that the treatment has been effective. c) Check the equipment. d) Contact the physician to review the care plan.

Answer: C-check the equipment

Which of the following terms will the nurse use when referring to blindness in the right or left halves of the visual fields of both eyes? a) Scotoma b) Nystagmus c) Homonymous hemianopsia d) Diplopia

Answer: C-homonymous hemianopsia

A 15-year-old baseball player was fielding a ground ball when it bounced and struck him in the left eye, leaving a large ecchymosis and edema. In your client education, after applying an ice pack, you explain to the client the functions of the various structures of the eye. What glands, contained in the eyelids, produce tears? a) Sebaceous b) Sweat c) Lacrimal d) Olfactory

Answer: C-lacrimal

Which of the following medications is the most effective agent in the treatment of Parkinson's disease (PD)? a) Amantadine (Symmetrel) b) Bromocriptine mesylate (Parlodel) c) Levodopa (Larodopa) d) Benztropine (Cogentin)

Answer: C-levadopa

The nurse is assessing an older client's vision. The nurse integrates knowledge of which of the following during the assessment? a) The skin around the eyes will be more elastic. b) The depth of the eyeball will be increased, leading to myopia. c) The power of the lens to accommodate will be decreased. d) Increased fat will be around the orbit.

Answer: C-the power of the lends to accommodate will be decreased

Which disturbance results in loss of half of the visual field? a) Anisocoria b) Nystagmus c) Diplopia d) Homonymous hemianopsia

Answer: D-Homonymous hemianopsia (loss of half the visual field)

The nurse working on a neurological unit is mentoring a nursing student. The student asks about a client who has sustained a primary and and secondary brain injury. The nurse correctly tells the student which of the following, related to the primary injury? a) It results from inadequate delivery of nutrients and oxygen to the cells. b) It refers to the difficulties suffered by the client and family related to the changes in the client. c) It refers to the permanent deficits seen after the rehabilitation process. d) It results from initial damage to the brain from the traumatic event.

Answer: D-It results from initial damage to the brain from the traumatic event.

A nurse is reviewing a CT scan of the brain, which states that the client has arterial bleeding with blood accumulation above the dura. Which of the following facts of the disease progression is essential to guide the nursing management of client care? a) The crash cart with defibrillator is kept nearby. b) Bleeding continues into the intracerebral area. c) Symptoms will evolve over a period of 1 week. d) Monitoring is needed as rapid neurologic deterioration may occur.

Answer: D-Monitoring is needed as rapid neurologic deterioration may occur.

The geriatric advanced practice nurse (APN) is doing client teaching with a client who has had a cerebrovascular accident (CVA) and the client's family. One concern the APN addresses is a potential for falls related to the CVA and resulting muscle weakness. What would be most important for the APN to include in teaching of the client and family related to this concern? a) Leg exercises to strengthen muscle weakness. b) Use of tripod cane. c) Need for support group due to decreased self image related to restricted mobility. d) Remove throw rugs and electrical cords from home environment.

Answer: D-Remove throw rugs and electrical cords from home environment.

Which of the following terms refers to surgical repair of the tympanic membrane? a) Myringotomy b) Tympanotomy c) Ossiculoplasty d) Tympanoplasty

Answer: D-Tympanoplasty

Glenn Middleton, a 67-year-old firefighter with a neurologic deficit, is informed that he will be transferred to a nursing home because his son is unable to care for him at home. While receiving a bed bath, the patient yells at the nurse, "You don't know what you are doing!" What is the best reaction by the nurse? a) Discontinue the bath and resume it later. b) Request that the patient be cared for by another nurse. c) Explain that he is getting good care. d) Accept the patient's behavior and do not take it personally.

Answer: D-accept the patient's behavior & do not take it personally

A patient with neurological infection develops cerebral edema from syndrome of inappropriate antidiuretic hormone (SIADH). Which of the following is an important nursing action for this patient? a) Administering prescribed antipyretics b) Maintaining adequate hydration c) Hyperoxygenation before and after tracheal suctioning d) Restricting fluid intake and hydration

Answer: D-restricting fluid intake& hydration

An osmotic diuretic, such as mannitol, is given to the patient with increased intracranial pressure (IICP) for which of the following therapeutic effects? a) To lower uncontrolled fevers b) To reduce cellular metabolic demands c) To increase urine output d) To dehydrate the brain and reduce cerebral edema

Answer: D-to dehydrate the brain & reduce cerebral edema

As a nurse practicing within a pediatric medicine group, you take your role quite seriously in preserving children's hearing and preventing hearing loss in your clients. What can you do to maintain hearing within your client base? a) Distribute earplugs to all clients. b) Prevent fevers. c) Reduce frequency and severity of ear infections. d) Increase antibiotic therapy use.

Answer: c-Reduce frequency and severity of ear infections.

The nurse is caring for a patient immediately following supratentorial intracranial surgery. What action by the nurse is appropriate? a) Place patient in the Trendelenburg position. b) Place patient in prone position with head turned to unaffected side. c) Place patient in the dorsal recumbent position. d) Place patient in supine position with head slightly elevated.

Answer: d-Place patient in supine position with head slightly elevated.

When performing the Weber test, where would the nurse place the vibrating tuning fork? Identify the area on the accompanying figure.

Answer: top of the forehead

Bell's palsy is a disorder of which cranial nerve? a) Trigeminal (V) b) Facial (VII) c) Vestibulocochlear (VIII) d) Vagus (X)

Answer:B-Facial (VII)

Which of the following nursing interventions is appropriate for a patient with double vision in the right eye due to MS? a) Apply an eye patch to the right eye. b) Administer eye drops as needed. c) Exercise the right eye twice a day (BID). d) Place needed items on the right side.

Apply an eye patch to the right eye.

A nurse is preparing to assess the cranial nerves of a client. The nurse is about to test CN I. What would the nurse do?

Ask a client to identify scents.

High doses of which of the following medications can produce bilateral tinnitus? a) Dramamine b) Antivert c) Promethazine d) Aspirin

Aspirin

CN V

Assess pain and light touch

Communication

Assessed by testing the frontal lobe

A 45-year-old client is admitted to the facility with excruciating paroxysmal facial pain. He reports that the episodes occur most often after feeling cold drafts and drinking cold beverages. Based on these findings, the nurse determines that the client is most likely suffering from which neurologic disorder? a) Migraine headache b) Trigeminal neuralgia c) Angina pectoris d) Bell's palsy

B

A client has short-term memory loss. To help the client cope with memory loss, the nurse should: a) tell the client in the morning what activities will be expected to be performed that day. b) place a single-date calendar where the client can view it. c) ask the client to try harder to remember things. d) instruct family members to ignore the behavior.

B

A client is undergoing testing to confirm a diagnosis of myasthenia gravis. The nurse explains that a diagnosis is made if muscle function improves after the client receives an I.V. injection of a medication. What is the medication the nurse tells the client he'll receive during this test? a) Cyclosporine b) Edrophonium c) Azathioprine d) Immunoglobulin G

B

A client who is prescribed by the health care provider (HCP) to take aspirin daily in order to prevent thrombus formation reports having ringing in the ears. The nurse advises the client to take which measure? a) Use acetaminophen instead. b) Contact the HCP. c) Increase fluid intake. d) Stop taking the aspirin.

B

A client with a head injury regains consciousness after several days. When the client first awakes, what should the nurse say to the client? a) "Can you tell me your name and where you live?" b) "You are in the hosipital. You were in an accident and unconscious." c) "I will get your family." d) "I will bet you are a little confused right now."

B

A nurse is assessing a client with meningitis. The nurse places the client in a supine position and flexes the client's leg at the hip and knee. The nurse notes resistance when straightening the knee and the client reports pain. The nurse should document what neurologic sign as positive? a) Lichtheim's sign b) Kernig's sign c) Babinski's reflex d) Brudzinski's sign

B

An older adult has vertigo accompanied with tinnitus as the result of Ménière's disease. The nurse should instruct the client to restrict which dietary element? a) protein b) sodium c) fluids d) potassium

B

Assessment of a client taking a nonsteroidal anti-inflammatory drug (NSAID) for pain management should include specific questions regarding which body system? a) cardiac b) gastrointestinal c) pulmonary d) renal

B

The client arrives in the emergency department following a bicycle accident in which the client's forehead hit the pavement. The client is diagnosed as having a hyphema. The nurse should place the client in which position? a) side-lying on the affected side b) semi-Folwer's c) side-lying on the unaffected side d) supine

B

The nurse enters the client's room as the client, who is sitting in a chair, begins to have a seizure. The nurse should first: a) restrain the client's body movements. b) ease the client to the floor. c) insert an airway into the client's mouth. d) lift the client onto his bed.

B

The unconscious client is to receive 200 mL of tube feeding every 4 hours. The nurse checks for the client's gastric residual before administering the next scheduled feeding and obtains 40 mL of gastric residual. The nurse should: a) dispose of the residual and continue with the feeding. b) readminister the residual to the client and continue with the feeding. c) withhold the tube feeding and notify the health care provider (HCP). d) delay feeding the client for 1 hour and then recheck the residual.

B

To encourage adequate nutritional intake for a client with Alzheimer's disease, a nurse should: a) fill out the menu for the client. b) stay with the client and encourage him to eat. c) give the client privacy during meals. d) help the client fill out his menu.

B

What should the nurse do first>/b> when a client with a head injury begins to have clear drainage from the nose? a) Administer an antihistamine for postnasal drip. b) Collect the drainage. c) Compress the nares. d) Tilt the head back.

B

When assessing the client with Parkinson's disease, the nurse should observe the client for: a) aphasia. b) a stiff, masklike facial expression. c) dry mouth. d) an exaggerated sense of euphoria.

B

Which is an initial sign of Parkinson's disease? a) akinesia b) tremor c) bradykinesia d) rigidity

B

Which statement indicates the client understands the expected course of Ménière's disease? a) "Continued medication therapy will cure the disease." b) "Control of the episodes is usually possible, but a cure is not yet available." c) "The disease process will gradually extend to the eyes." d) "Bilateral deafness is an inevitable outcome of the disease."

B

The nurse has positioned a client supine and asked her to perform the heel-to-shin test. An inability to run each heel smoothly down each shin should prompt the nurse to perform further assessment in what domain?

Balance and coordination

A nurse cares for a client who suffered a cerebrovascular accident and demonstrates the inability to speak clearly. The nurse recognizes that injury has occurred to what portion of the brain?

Broca's area

A client accidentally splashes chemicals into one eye. The nurse knows that eye irrigation with plain tap water should begin immediately and continue for 15 to 20 minutes. What is the primary purpose of this first-aid treatment? a) To serve as a stopgap measure until help arrives b) To hasten formation of scar tissue c) To prevent vision loss d) To eliminate the need for medical care

C

A client experienced a stroke that damaged the hypothalamus. The nurse should anticipate that the client will have problems with: a) thinking and reasoning. b) visual acuity. c) body temperature control. d) balance and equilibrium.

C

A client has been diagnosed with a basal skull fracture following a motor vehicle accident and now presents with increasing drowsiness and is febrile. The nurse knows that the client is most at risk for developing which of the following? a) Paralytic ileus b) Renal failure c) Meningitis d) Pneumonia

C

A client is admitted with a cervical spine injury sustained during a diving accident. When planning this client's care, the nurse should assign highest priority to which nursing diagnosis? a) Dressing or grooming self-care deficit b) Disturbed sensory perception (tactile) c) Ineffective breathing pattern d) Impaired physical mobility

C

A client is diagnosed with a brain tumor. The nurse's assessment reveals that the client has difficulty interpreting visual stimuli. Based on these findings, the nurse suspects injury to which lobe of the brain? a) Parietal b) Temporal c) Occipital d) Frontal

C

A client was running along an ocean pier, tripped on an elevated area of the decking, and struck his head on the pier railing. According to his friends, "He was unconscious briefly and then became alert and behaved as though nothing had happened." Shortly afterward, he began complaining of a headache and asked to be taken to the emergency department. If the client's intracranial pressure (ICP) is increasing, the nurse should expect to observe which sign first? a) Pupillary asymmetry b) Irregular breathing pattern c) Declining level of consciousness (LOC) d) Involuntary posturing

C

A client who is paraplegic cannot feel the lower extremities and has been positioned on the side. The nurse should inspect which area that is a potential pressure point when the client is in this position? a) sacrum b) occiput c) ankles d) heel

C

A client with a head injury regains consciousness after several days. When the client first awakes, what should the nurse say to the client? a) "I will get your family." b) "I will bet you are a little confused right now." c) "You are in the hosipital. You were in an accident and unconscious." d) "Can you tell me your name and where you live?"

C

A nurse is caring for a client with dementia. A family member of the client asks what the most common cause of dementia is. Which response by the nurse is most appropriate? a) "Depression may manifest as dementia in elderly clients." b) "Drug interactions are the most common cause of dementia in the elderly." c) "The most common cause of dementia in the elderly is Alzheimer's disease." d) "Dementia is a terrible disease of the elderly."

C

A nurse on a neurologic unit is working on performance improvement with a stroke-management team. The nurse identifies a gap between the time a client enters the emergency department (ED) and the time he's admitted to the intensive care unit (ICU) for aggressive treatment. She's meeting with the team to develop a change strategy using indicators. Which statement by a team member indicates a need for further teaching regarding performance management? a) "We can collaborate with staff from the ED and the ICU to formulate strategies and implement change." b) "We can review ED staffing to see if shortages affect ICU admission." c) "We can discipline the ED staff for not getting the clients to the ICU fast enough." d) "We can use statistics gathered in the ED during triage to determine the average time for admission to the ICU."

C

A nurse, a licensed practical nurse (LPN), and a nursing assistant are caring for a group of clients. The nurse asks the nursing assistant to check the pulse oximetry level of a client who underwent laminectomy. The nursing assistant reports that the pulse oximetry reading is 89%. The client Kardex contains an order for oxygen application at 2 L/min should the pulse oximetry level fall below 92%. The nurse is currently assessing a postoperative client who just returned from the postanesthesia care unit. How should the nurse proceed? a) Complete the assessment of the new client before attending to the client who underwent laminectomy. b) Immediately go the client's room and assess vital signs, administer oxygen at 2 L/minute, and notify the physician. c) Ask the LPN to obtain vital signs and administer oxygen at 2 L/min to the client who underwent laminectomy. d) Ask the nursing assistant to notify the physician of the low pulse oximetry level.

C

The nurse has asked the unlicensed assistive personnel (UAP) to ambulate a client with Parkinson's disease. The nurse observes the UAP pulling on the client's arms to get the client to walk forward. The nurse should: a) assist the UAP with getting the client back in bed. b) have the UAP keep a steady pull on the client to promote forward ambulation. c) explain how to overcome a freezing gait by telling the client to march in place. d) give the client a muscle relaxant.

C

The nurse is assessing the level of consciousness for a client who just had open heart surgery. When asked, the client can give his name but is not sure about where he is or the time of day. What should the nurse do? a) Notify the surgeon. b) Encourage the client's wife to orient the client. c) Tell the client where he is and the time of day. d) Rub the client's sternum to arouse the client.

C

The nurse is instructing a client with Ménière's disease how to recognize vertigo. The nurse should tell the client to notice: a) light-headedness. b) an episode of blackout. c) a feeling that the environment is in motion. d) narrowed vision preceding fainting.

C

What assessment findings would indicate an emergency myasthenia crisis? a) Impairment of functioning of the autonomic and skeletal muscles b) Airway obstruction, profound muscle weakness, and inability to move c) Severe dyspnea, intensification of dysphagia, and dysarthria d) Paralysis of the muscles and hyperventilation

C

Which technique is appropriate when the nurse is irrigating an adult client's ear to move cerumen? a) After instilling the solution, pack the ear canal tightly with a cotton ball. b) Use sterile solution and equipment. c) Allow the irrigating solution to run down the wall of the ear canal. d) Make sure the irrigating solution is cool.

C

During the health history, a client reports a decrease in his ability to smell. During the physical assessment, the nurse would make sure to assess which cranial nerve?

CN I

The husband of a 65-year-old female tells the nurse, "My wife is having trouble navigating the steps in our home and she needs my help to step down off a curb." What part of the nervous system should the nurse assess for a potential source of the problem?

Cerebellum

The nurse is performing the Romberg test. Which of the following indicate a normal finding? -Client stands erect with minimal swaying -Client sways when eyes are closed -Client prevents himself from falling -Client maintains balance when walking

Client maintains balance when walking

What should the nurse assess to test the function of the frontal lobe?

Communication

The nurse walks into a client's room and finds that the client is disoriented to time and place but is awake and responsive. What term best describes this patient?

Confused

Diencephalon

Consists of thalamus and hypothalamus

Cerebrospinal fluid

Cushions CNS, provides nourishment to CNS, and removes wastes

A client who has been treated for chronic open-angle glaucoma (COAG) for 5 years asks the nurse, "How does glaucoma damage my eyesight?" The nurse's reply should be based on the knowledge that COAG: a) is caused by decreased blood flow to the retina. b) results from chronic eye inflammation. c) leads to detachment of the retina. d) causes increased intraocular pressure.

D

A client with a suspected overdose of an unknown drug is admitted to the emergency department. Arterial blood gas values indicate respiratory acidosis. What should the nurse do first? a) Monitor the client's heart rhythm. b) Prepare for gastric lavage. c) Obtain a urine specimen for drug screening. d) Prepare to assist with ventilation.

D

A client with a tentative diagnosis of myasthenia gravis is admitted for a diagnostic workup. Myasthenia gravis is confirmed by: a) Kernig's sign. b) a positive sweat chloride test. c) Brudzinski's sign. d) a positive edrophonium test.

D

A client with an inflammatory ophthalmic disorder has been receiving repeated courses of a corticosteroid ointment, one-half inch in the lower conjunctival sac four times a day as directed. The client reports a headache and blurred vision. The nurse suspects that these symptoms represent: a) common adverse reactions to corticosteroid therapy. b) incorrect ointment application. c) expected drug effects that should diminish over time. d) increased intraocular pressure (IOP).

D

A client with quadriplegia is in spinal shock. What finding should the nurse expect? a) Positive Babinski's reflex along with spastic extremities b) Hyperreflexia along with spastic extremities c) Spasticity of all four extremities d) Absence of reflexes along with flaccid extremities

D

A nurse caring for a client who had a stroke is using the unit's new computerized documentation system. The nurse uses the information technology appropriately when she: a) determines a client's identity from a computer chart. b) documents medications before administration. c) e-mails information about a client to a friend at home. d) documents medications after administration.

D

A nurse completing her management rotation in the intensive care unit (ICU) is working with an experienced ICU nurse. One client's work supervisor calls to "check up" on the client. The nurse offers to transfer the call to the client's family members. The experienced ICU nurse recognizes this action as: a) passing the buck to avoid work. b) a violation of privacy laws. c) a clever way of avoiding the supervisor. d) protection of the client's privacy.

D

A nurse is assisting during a lumbar puncture. How should the nurse position the client for this procedure? a) Prone, with the head turned to the right b) Supine, with the knees raised toward the chest c) Lateral, with right leg flexed d) Lateral recumbent, with chin resting on flexed knees

D

A nurse is caring for a client admitted to the unit with a seizure disorder. The client seems upset and asks the nurse, "What will they do to me? I'm scared of the tests and of what they'll find out." The nurse should focus her teaching plans on which diagnostic tests? a) Electrocardiography, TEE, prothrombin time (PT), and International Normalized Ratio (INR) b) Transesophageal echocardiogram (TEE), troponin levels, and a complete blood count c) X-ray of the brain, bone marrow aspiration, and EEG d) EEG, blood cultures, and neuroimaging studies

D

As a first step in teaching a woman with a spinal cord injury and quadriplegia about her sexual health, the nurse assesses her understanding of her current sexual functioning. Which statement by the client indicates she understands her current ability? a) "I cannot have sexual intercourse because it causes hypertension, but other sexual activity is okay." b) "I will not be able to have sexual intercourse until the urinary catheter is removed." c) "I should be able to participate in sexual activity, but I will be infertile." d) "I can participate in sexual activity but might not experience orgasm."

D

Complications associated with a tracheostomy tube include: a) pneumothorax. b) acute respiratory distress syndrome (ARDS). c) decreased cardiac output. d) damage to the laryngeal nerve.

D

Sodium polystyrene sulfonate is prescribed for a client following crush injury. The drug is effective if: a) the pulse is weak and irregular. b) the ECG is showing tall, peaked T waves. c) there is muscle weakness on physical examination. d) the serum potassium is 4.0 mEq/L (4.0 mmol/L).

D

The client has a sustained increased intracranial pressure (ICP) of 20 mm Hg. Which client position would be most appropriate? a) left Sims position b) the head elevated on two pillows c) Trendelenburg's position d) the head of the bed elevated 15 to 20 degrees

D

To assess a client's cranial nerve function, a nurse should assess: a) arm drifting. b) hand grip. c) orientation to person, time, and place. d) gag reflex.

D

When the nurse talks with a client with multiple sclerosis who has slurred speech, which nursing intervention is contraindicated? a) encouraging the client to speak slowly b) asking the client to repeat indistinguishable words c) encouraging the client to speak distinctly d) asking the client to speak louder when tired

D

Which action is contraindicated for a client with seizure precautions? a) allowing the client to wear his or her own clothing b) encouraging the client to perform his or her own personal hygiene c) encouraging the client to be out of bed d) assessing the client's oral temperature with a glass thermometer

D

Which activity should the nurse encourage the client to avoid when there is a risk for increased intracranial pressure (ICP)? a) turning b) deep breathing c) passive range-of-motion (ROM) exercises d) coughing

D

Which statement indicates that a client understands the nurse's teaching about phenytoin for the diagnosis of seizures? a) "I will only be on this type of medication for a short while." b) "This medication can help reduce my anxiety." c) "This medication may keep me awake." d) "This medication will not cure my disease."

D

When assessing deep tendon reflexes in an elderly client what finding would the nurse anticipate?

Decreased reaction time

What would the nurse most likely find when assessing a client diagnosed with a frontal lobe contusion following a motor vehicle accident?

Difficulty speaking

The nurse has assessed pupil size in a newly admitted client on the neuro-trauma unit. The client's pupils are unequal in size, and the healthcare provider is notified. The nurse is instructed that the findings indicate physiological anisocoria based on the healthcare provider's knowledge of the client's history. What is the nurse's best action?

Document the findings and healthcare provider's response.

A client with Parkinson's disease has been receiving levodopa as treatment for the past 7 years. The client comes to the facility for an evaluation and the nurse observes facial grimacing, head bobbing, and smacking movements. The nurse interprets these findings as which of the following? a) Dysphonia b) Micrographia c) Bradykninesia d) Dyskinesia

Dyskinesia

Psychiatric medication

Dystonia usually due to this type of medication

Which of the following terms refer to a method of recording, in graphic form, the electrical activity of the muscle?

Electromyogram Electromyogram is a method of recording, in graphic form, the electrical activity of the muscle. Electroencephalogram is a method of recording, in graphic form, the electrical activity of the brain. Electrocardiography is performed to assess the electrical activity of the heart. Electrogastrography is an electrophysiologic study performed to assess gastric motility disturbances.

Cranial Nerve VII

Facial: Sensory & Motor Sensory: contains sensory fibers for taste on anterior 2/3 of tongue, and stimulates salivary secretions and tears from lacrimal glands. (saliva and tears) Motor: supplies the facial muscles and affects facial expressions

During assessment, the nurse notes the client has limited movement of his lower extremities and sways when standing with feet together. The nurse identifies that the client is at risk for what?

Falls

T/F After testing deep tendon reflexes, the nurse documents 2+. The nurse should evaluate further.

False

Cranial Nerve IX

Glossopharyngeal: Sensory & Motor Sensory: taste on posterior 1/3 of tongue; gag reflex Motor: provides secretory fibers to the parotid salivary glands; promotes swallowing

The nurse should monitor for which of the following manifestations in a patient who has had LASIK surgery? a) Excessive tearing b) Stye formation c) Cataract formation d) Halos and glare

Halos & glare

The nurse is preparing to assess balance in an older adult client. Which test would the nurse plan on possibly omitting from the exam?

Hop on one foot

Cranial Nerve XII

Hypoglossal: Motor innervates tongue muscles that promote movement of food and talking.

Which part of the brain controls the vital functions of temperature, heart rate, blood pressure, sleep, the anterior and posterior pituitary, the autonomic nervous system, and emotions and maintains overall autonomic control?

Hypothalamus

The nurse assesses the motor system as part of the full neurological examination. In order to effectively assess this system, which of the following instructions should be given to the client? -Instruct the client to state the current date and place -Instruct the client to flex and extend the right elbow -Instruct the client to smile -Ask the client to close the eyes

Instruct the client to flex and extend the right elbow

Function of the Parietal Lobe

Interprets tactile sensations, including touch, pain, temp, shapes, and two-point discrimination

You are caring for a client admitted with a stroke. Imaging studies indicate an embolus partially obstructing the right carotid artery. What type of stroke do you know this client has? a) Ischemic b) Hemorrhagic c) Right-sided d) Left-sided

Ischemic

What does a neuro check assess?

Level of consciousness pupillary checks movement and strength of extremities vital signs

The nurse is caring for a patient after lumbar puncture. The patient is complaining of a severe headache. Which of the following actions should the nurse complete? Select all that apply. Maintain the patient on bed rest. Administer fluids to the patient. Position the patient in the supine position. Prepare for an epidural blood patch. Administer analgesic medication.

Maintain the patient on bed rest. Administer fluids to the patient. Administer analgesic medication. When the patient assumes an upright position, tension and stretching of the venous sinuses and pain-sensitive structures occur. A postpuncture headache is usually managed by bed rest, analgesic agents, and hydration. Postlumbar puncture headache may be avoided if a small-gauge needle is used and if the patient remains prone after the procedure. When more than 20 mL of the CSF is removed, the patient is positioned supine for 6 hours.

When performing an assessment of the nervous system, it is most appropriate for a nurse to complete it in which sequence?

Mental status, cranial nerves, motor/cerebellar, sensory, reflexes

Which of the following is considered a central nervous system (CNS) disorder? a) Guillain-Barré b) Multiple sclerosis c) Myasthenia gravis d) Bell's palsy

Multiple sclerosis

Bicep reflex

Normal responce is elbow flexes and muscle contracts

Oculomotor nerve paralysis

Nurse observes client's pupils dilated to 6cm

Cranial Nerve III

Oculomotor: Motor contracts eye muscles to control eye movements, constricts pupils, and elevates eyelids

Cranial Nerve I

Olfactory Nerve: Sensory carries smell impulses from nasal mucous membrane to brain

Cranial Nerve II

Optic: Sensory carries visual impulses from eye to brain

Which of the following is the term for swelling of the optic disc due to increased IOP? a) Photophobia b) Ptosis c) Chemosis d) Papilledema

Papilledema

Which of the following is a late symptom of spinal cord compression? a) Urinary incontinence b) Paralysis c) Fecal incontinence d) Urinary retention

Paralysis

The nurse is assessing the client's pupils following a sports injury. Which of the following assessment findings indicates a neurologic concern? Select all that apply. Unequal pupils Pupil reaction quick Pinpoint pupils Absence of pupillary response Pupil reacts to light

Pinpoint pupils Absence of pupillary response Unequal pupils

The nurse is caring for a patient immediately following supratentorial intracranial surgery. What action by the nurse is appropriate? a) Place patient in supine position with head slightly elevated. b) Place patient in prone position with head turned to unaffected side. c) Place patient in the Trendelenburg position. d) Place patient in the dorsal recumbent position.

Place patient in supine position with head slightly elevated.

A client has sustained nerve damage as a result of an automobile accident and has lost the ability to sense position, vibration, and fine touch. Which neural pathway should the nurse suspect to be damaged?

Posterior columns

The nurse is reviewing the medical record of a client with glaucoma. Which of the following would alert the nurse to suspect that the client was at increased risk for this disorder? a) Prolonged use of corticosteroids b) Hyperopia since age 20 years c) History of respiratory disease d) Age younger than 40 years

Prolonged use of corticosteroids

A client has undergone surgery for a spinal cord tumor that was located in cervical area. The nurse would be especially alert for which of the following? a) Hemorrhage b) Respiratory dysfunction c) Bowel incontinence d) Skin breakdown

Respiratory dysfunction

A client's patellar reflex is normal for the right side but diminished on the left. Using the scale for grading reflexes, how should the nurse document this finding?

Right knee +2; Left knee +1

Spinothalamic tract

Sensations of pain, temperature, and crude and light touch travel by way of the spinothalamic tract

Aura

Should be assessed for if a client is admitted with a seizure disorder

CN XI

Shoulder shrugging test

Cranial Nerve XI

Spinal Accessory: Motor Innervates neck muscles that promote movement of shoulders and head rotation. Promotes some movements of larynx

A nurse is with a client who is victim of a shooting. The client has an increased pulse rate and pupil dilation and is clearly in stress. The nurse recognizes the "fight-or-flight" response in this client and understands that this represents an activation of which system?

Sympathetic nervous system

CN VIII

Test the client's hearing for lateralization and bone and air conduction

Which of the following assessments is most likely to provide insight into the function of the client's CN VIII?

Test the client's hearing for lateralization and bone and air conduction.

CN I

Tested by client's ability to identify smells

A client is visiting the health care facility for follow-up care for a stroke. Today he has increased muscle tone, some involuntary movements, an abnormal gait, and a slowness of response in movements. He most likely has involvement of which of the following?

The basal ganglia

A nurse and nursing student are caring for a client recovering from a lumbar puncture yesterday. The client reports a headache despite being on bedrest overnight. The physician plans an epidural blood patch this morning. The student asks how this will help the headache. The correct reply from the nurse is which of the following?

The blood will seal the hole in the dura and prevent further loss of cerebral spinal fluid. Loss of CSF causes the headache. Occasionally, if the headache persists, the epidural blood patch technique may be used. Blood is withdrawn from the antecubital vein and injected into the site of the previous puncture. The rationale is that the blood will act as a plug to seal the hole in the dura and preven further loss of CSF. The blood is not put into the subarachnoid space. The needle is inserted below the level of the spinal cord, which prevents damage to the cord. It is not a lack of moisture that prevents healing; it is more related to the size of the needle used for the puncture.

The nurse is performing the Romberg test as part of a client's focused neurological assessment. What finding would constitute a positive Romberg test?

The client moves her feet apart to prevent herself from falling.

A client is hospitalized when they present to the Emergency Department with right-sided weakness. Within 6 hours of being admitted, the neurologic deficits had resolved and the client was back to their presymptomatic state. The nurse caring for the client knows that the probable cause of the neurologic deficit was what? a) Cerebral aneurysm b) Right-sided stroke c) Transient ischemic attack d) Left-sided stroke

Transient ischemic attack

Intentional tremors

Tremors the occur with intentional movements

Cranial Nerve V

Trigeminal: Sensory & Motor carries sensory impulses of pain, touch, and temp from the face to the brain. Influences biting and chewing

Cranial Nerve IV

Trochlear: Motor contracts one eye muscle to control inferomedial eye movement

Acoustic neuromas are benign tumors of which of the following cranial nerves? a) V b) VII c) VIII d) VI

VIII

Cranial Nerve X

Vagus: Sensory & Motor carries sensations from the throat, larynx, heart, lungs, bronchi, GI tract, and abdominal viscera. Promotes swallowing, talking, and production of digestive juices.

When conducting a Romberg test, why does the nurse ask the client to stand feet together with eyes open and then closed?

Vision can compensate for loss of position sense.

Myasthenia Gravis

Weakness made worse with repeated effort and improved with rest

VIII

Whisper test, Rinne, and Weber

Neuro check

a brief screening of the client's neurologic status. Done in emergency situations and when frequent assessments are needed during an acute phase of illness to detect rapid changes in neurological status.

Corpus Callosum

a bundle of nerve fibers responsible for the communication between the right and left cerebral hemispheres

Brown-Sequard syndrome

a hemisection of the spinal cord resulting in ipsilateral loss of strength and proprioception and contralateral loss pf pain and temp

transient ischemic attack (TIA)

a mini-stroke that causes no damage but indicates stroke risk

A client with a tentative diagnosis of myasthenia gravis is admitted for a diagnostic workup. Myasthenia gravis is confirmed by: a) a positive edrophonium (Tensilon) test. b) Kernig's sign. c) a positive sweat chloride test. d) Brudzinski's sign.

a positive edrophonium (Tensilon) test

The nurse is assessing the neurologic system of an adult client. To test the client's use of memory to learn new information, the nurse should ask the client a) "Can you repeat brown, chair, textbook, tomato?" b) "Can you repeat rose, hose, nose, clothes?" c) "What did you have for breakfast?" d) "How old were you when you began working?"

a) "Can you repeat brown, chair, textbook, tomato?"

A nurse observes a client's gait and notes it to be wide based and staggering. The Romberg test results were positive. The nurse recognizes this as what type of abnormal gait? a) Cerebellar ataxia b) Spastic hemiparesis c) Parkinsonian gait d) Footdrop gait

a) Cerebellar ataxia

The husband of a 65-year-old female tells the nurse, "My wife is having trouble navigating the steps in our home and she needs my help to step down off a curb." What part of the nervous system should the nurse assess for a potential source of the problem? a) Cerebellum b) Deep tendon reflexes c) Temporal lobe d) Cranial nerves

a) Cerebellum

The nurse performing an admission assessment on an older adult. What would be an expected finding? a) Decreased vision b) Delirium c) Numbness and tingling d) Dizziness

a) Decreased vision

Upon reviewing the client's medical record, the nurse finds the client has left ptosis. The nurse would assess the client for what? a) Drooping of the left eye b) Swelling of the optic nerve c) Loss of visual fields on the left d) Drooping of the left side of the mouth

a) Drooping of the left eye

A nurse assesses a client for pupillary response of the eyes and finds a unilateral dilated pupil that is unresponsive to light or accommodation. The nurse recognizes that which cranial nerve is responsible for the damage of pupillary response? a) III b) I c) V d) II

a) III

When testing sensory function of the trigeminal nerve (CN V), which of the following sensations would the nurse assess? a) Pain and light touch b) Proprioception and extinction c) Dull touch and vibration d) Vibration and stereognosis

a) Pain and light touch

Lifestyle can play a big part in developing risk factors for stroke. Which of the following can greatly reduce a client's risk for stroke? Select all that apply. a) Regularly exercising b) Maintaining a healthy weight c) Following a sedentary lifestyle d) Eating a high-sodium diet e) Quitting smoking

a) Regularly exercising b) Maintaining a healthy weight e) Quitting smoking

The nurse is doing a neurologic screening examination. The nurse should include some aspect of which areas? Select all that apply. a) motor system b) sensory system c) cranial nerves d) mental status e) reflexes f) cardiovascular system

a) motor system b) sensory system c) cranial nerves d) mental status e) reflexes

A patient is in the emergency room with what could be a lumbar injury. Which deep tendon reflex would be most appropriate to test? a) patellar b) supinator c) triceps d) ankle

a) patellar

The nurse is preparing to perform the Romberg test on an adult male client. The nurse should instruct the client to a) stand erect with arms at the sides and feet together. b) keep his eyes open while he bends at the knees. c) squat down as far as he is able to do so. d) touch the tip of his nose with his finger.

a) stand erect with arms at the sides and feet together.

testing occipital lobe

ability to read

intention tremor

absent at rest, appear with activity and often get worse as the target is neared.

The nurse is assessing the neurologic system of an adult client. To test the client's motor function of the facial nerve, the nurse should

ask the client to purse the lips.

Diabetic peripheral neuropathy

associated with loss of sensation, burning, and tingling in the feet that often radiate up

athetosis

athetoid movements are slower and more twisting and writhing than choreiform movements, and have a larger amplitude. most commonly involve the face and distal extremities. causes include cerebral palsy

The nurse is caring for a client in the hospital and identifies the client to be experiencing acute confusion after cardiac surgery. The nurse recognizes this as what? a) Dementia b) Delirium c) Hypoxia d) Amnesia

b) Delirium

The nurse is preparing to assess balance in an older adult client. Which test would the nurse plan on possibly omitting from the exam? a) Tandem walking b) Hop on one foot c) Gait d) Romberg

b) Hop on one foot

When performing an assessment of the nervous system, it is most appropriate for a nurse to complete it in which sequence? a) Cranial nerves, motor/cerebellar, sensory, reflexes, mental status b) Mental status, cranial nerves, motor/cerebellar, sensory, reflexes c) Motor/cerebellar, sensory, reflexes, cranial nerves, mental status d) Reflexes, sensory, motor/cerebellar, cranial nerves, mental status

b) Mental status, cranial nerves, motor/cerebellar, sensory, reflexes

Mr. Clyde presents at the clinic with a complaint of weakness that is made worse with repeated effort and improves with rest. Mr. Clyde's complaint is consistent with what health problem? a) Parkinson disease b) Myasthenia gravis c) Ischemic stroke d) Lyme disease

b) Myasthenia gravis

You are admitting a patient new to the clinic who states, "My face feels funny." When you assess the patient you find isolated facial sensory loss to pain and no neurologic deficits in his extremities. What would you expect this patient to be diagnosed with? a) Stroke b) Trigeminal neuralgia c) Horner syndrome d) Bell palsy

b) Trigeminal neuralgia

The nurse plans to test which cranial nerve when testing an elderly patient's hearing status? a) V b) VIII c) VII d) VI

b) VIII

Tics

brief, repetitive, stereotyped, coordinated movements occurring at irregular intervals. Examples: repetitive winking, grimacing, and shoulder shrugging.

What should the nurse assess to test the function of the occipital lobe? a) Tactile sensation b) Impulses from the ear c) Ability to read d) Communication

c) Ability to read

The nurse has completed a Glasgow Coma Scale assessment and assigns the client a score of three. Which is the best way for the nurse to assess pain in this client? a) Utilize the FACES scale. b) Clients assigned this low score are pain free. c) Assess for nonverbal signs. d) Use a verbal 0-10 rating scale.

c) Assess for nonverbal signs.

While conversing with a 42-year-old client, the nurse notes the client's tendency to repeatedly wink and shrug his shoulders at irregular intervals. The movements do not appear to correlate with the client's conversation. How should the nurse best follow up this observation? a) Assess the client's cranial nerves VIII, IX, and X. b) Assess the client's immunization history. c) Assess the client's medication regimen and history of recreational drug use. d) Order a CT (computed tomography) of the client's head.

c) Assess the client's medication regimen and history of recreational drug use.

The nurse working in the emergency department is assessing an intoxicated driver involved in a motor vehicle crash when the client insists on ambulating to the bathroom. The nurse escorts the client and calls for help while anticipating which abnormal gait in this client that places him at risk for falls? a) Sensory ataxia b) Scissors movement c) Cerebellar ataxia d) Spastic Hemiparesis

c) Cerebellar ataxia

A client has sustained an injury to the cerebellum. Which area would be the primary area for assessment? a) Neurologic system b) Cardiac function c) Coordination d) Vital signs

c) Coordination

Which part of the brain controls the vital functions of temperature, heart rate, blood pressure, sleep, the anterior and posterior pituitary, the autonomic nervous system, and emotions and maintains overall autonomic control? a) Medulla b) Cerebral cortex c) Hypothalamus d) Brain stem

c) Hypothalamus

A nurse is reviewing a client's health record while interviewing her. The nurse sees in the patient's record a score of 3+ on the biceps reflex test from her previous visit. The nurse understands that this finding indicates which of the following? a) Present but decreased b) Normal c) Increased or brisk, but not pathologic d) Exaggerated; indicator of possible upper motor neuron lesion

c) Increased or brisk, but not pathologic

A client with a history of seizure disorder and taking several seizure medications reports that a friend noted "jumping eye movements." The client describes a sensation of movement at rest since his medications were adjusted upward following a breakthrough seizure several weeks ago. Examination shows that both eyes slowly move to the right then quickly jump to the left. Based on these signs, which of the following is true? a) This most likely has an ominous cause. b) This is called sacchadic eye movement. c) This is called nystagmus to the left. d) This represents a subclinical seizure.

c) This is called nystagmus to the left.

Which of the following assessment techniques should the nurse use to determine a client's stereognosis? a) With the client's eyes closed, trace a number on the client's hand and ask him or her to identify the number. b) Briefly touch a point on the client's skin and ask the client to open his or her eyes and point to the place touched. c) With the client's eyes closed, place a coin or key in hand and ask him or her to identify the object. d) Using two ends of an open paper clip, touch two points on the client's finger pad simultaneously and identify the minimal distance that the client can discriminate between the points.

c) With the client's eyes closed, place a coin or key in hand and ask him or her to identify the object.

A client is concerned about tripping when walking and feeling uncoordinated. Which part of the brain might be causing this client's symptoms?

cerebellum

footdrop

client lifts foot and knee high with each step, then slaps the foot down hard on the ground. client cannot walk on heels. characteristics of diseases of lower motor neurons.

functions of the spinal cord

conducts sensory impulses up ascending tracts to the brain, conducts motor impulses down descending tracts to neurons that stimulate glands and muscles throughout the body, and is responsible for simple reflex activity.

Function of the subarachnoid space

cushions the brain and spinal cords, nourishes the CNS, and removes waste materials

The nurse working in the emergency department is assessing an intoxicated driver involved in a motor vehicle crash when the client insists on ambulating to the bathroom. The nurse escorts the client and calls for help while anticipating which abnormal gait in this client that places him at risk for falls? a) Sensory ataxia b) Scissors movement c) Spastic Hemiparesis d) Cerebellar ataxia

d) Cerebellar ataxia

What is the level of the spinal cord associated with the knee (patellar) deep tendon reflex? a) T11 and T12 b) T9 and T10 c) S1 d) L2 to L4

d) L2 to L4

The nurse is preparing to percuss a client's reflexes in his arms. To use the reinforcement technique, the nurse should ask the client to a) hold his neck toward the floor. b) stretch the opposite arm. c) straighten his legs forward. d) clench his jaw.

d) clench his jaw.

The cranial nerve that has sensory fibers for taste and fibers that result in the "gag reflex" is the a) vagus. b) hypoglossal. c) trigeminal. d) glossopharyngeal.

d) glossopharyngeal.

Function of the Frontal Lobe

directs voluntary, skeletal actions. Also influences communication, emotions, intellect, reasoning ability, judgment, and behavior. Contains Broca's area which is responsible for speech

A nurse is planning care for a client who has been diagnosed with restless leg syndrome. Which intervention is the most effective for temporary relief of the symptoms?

exercising the legs

Level of consciousness (LOC) can be assessed based on criteria in the Glasgow Coma Scale (GCS). Which of the following indicators are assessed in the GCS? Select all that apply. a) Eye opening b) Muscle strength c) Intelligence d) Verbal response e) Motor response

eye opening, verbal response, motor response

spastic hemiparesis

flexed arm held close to body while client drags toe of leg or circles it stiffly outward and forward. seen with lesions of the upper motor neurons in the cortical spinal tract, such as occurs in strokes

CN VII

frowning and closing eyes

cardiovascular accident (CVA)

happens when blood flow to a portion of the brain is interrupted or stops

most important contributor to stroke

high blood pressure

Testing temporal lobe

impulses from the ear

Function of the Occipital Lobe

influences the ability to read with understanding an dis the primary visual receptor center

posterior cord syndrome

injury results in loss of proprioception and variable preservation of motor function and pain and temp responses

central cord syndrome

injury results in sparing and preferentially upper- more than lower-extremity weakness

anterior cord syndrome

injury results in variable loss of motor function as well as pain and temp.

deep reflexes depend on

intact sensory nerve, a functional synapse in the spinal cord, an intact motor nerve, a neuromuscular junction, and competent muscles

ischemic strokes

may be due to narrowing of the artery from clot formation or from a clot breaking off from another location in the brain or body, causing blockage as it lodges in the smaller brain artery.

gray matter

mediates higher-level functions such as memory, perception, communication, and initiation of voluntary movements. Rims the cerebral hemispheres, forming the cerebral cortex.

Resting (static) tremors

most prominent at rest, and may decrease or disappear with voluntary movement.

CN XII

move the tongue side to side

chorea choreiform movements of the hand

movements are brief, rapid, jerky, irregular, and unpredictable. They occur at rest or interrupt normal coordinated movements. Seldom repeat themselves. Face, head, lower arms, and hands are usually involved. Causes include Sydenham's chorea and Huntington's disease

hemorrhagic stroke

occurs when a vessel becomes weak and bursts

sensations that travel via the spinothalamic tract

pain, temperature, and crude and light touch

hypothalamus

part of the autonomic nervous system. Responsible for regulating many body functions including water balance, appetite, vital signs, sleep cycles, pain perception, and emotional status

sensations that travel via the posterior columns

position, vibration, and fine touch

cerebellum

primary functions include coordination and smoothing of voluntary movements, maintenance of equilibrium, and maintenance of muscle tone. Does not initiate movement.

reason for testing reflexes

provides clues to the integrity of deep and superficial reflexes

Reason for examining the sensory system.

provides info regarding the integrity of the spinothalamic tract, posterior columns of spinal cord, & parietal lobes of brain.

cranial nerve evaluation

provides info regarding the transmission of motor and sensory messages, primarily to the head and neck

mental status examination

provides information about cerebral cortex function.

function of the temporal lobe

receives and interprets impulses from the ear. contains Wernicke's area which is responsible for intrepreting auditory stimuli.

thalamus

screening and directing the impulses to specific areas in the cerebral cortex

A client says that an object placed in the hand is a pair of scissors when the object is a paper clip. Which aspect of the client's neurologic system should the nurse identify as being compromised?

sensory

the midbrain

serves as a relay center for ear and eye reflexes, and relays impulses between the higher cerebral centers and the lower pons, medulla, cerebellum, and spinal cord.

parkinsonian gait

shuffling gait, turns accomplished in very stiff manner. stooped-over posture with flexed knees and hips. typically seen in parkinson's disease and drug-induces parkinsonian b/c of effects on the basal ganglia.

superficial reflexes depend on

skin receptors rather than muscles

When the nurse is assessing the motor function of cranial nerve VII as part of the neurological examination, what should the nurse instruct the client to do?

smile

The Glasgow Coma Scale measures the level of consciousness in clients who are at high risk for rapid deterioration of the nervous system. A score of 13 indicates

some impairment.

scissors gait

stiff, short gait; thighs overlap each other with each step seen with partial paralysis of the legs

Testing parietal lobe

test for tactile sensation

What do cerebral abnormalities disturb?

the client's intellectual ability, communication ability, or emotional behaviors

postural tremor

these tremors appear when the affected part is maintaining a posture.

Why are the motor and cerebellar systems assessed?

to determine functioning of the pyramidal and extrapyramidal tracts

The nurse is planning to test position sensation in an adult female client. To perform this procedure, the nurse should ask the client to close her eyes while the nurse moves the client's

toes up or down.

Cerebellar ataxia

wide-based and staggering gait (Romberg will be positive) can be seen in persons with cerebellar disease or alcohol or drug intoxication

cerebellar ataxia

wide-based, staggering, unsteady gait. Romberg test results are positive. seen with cerebellar diseases or alcohol or drug intoxication


संबंधित स्टडी सेट्स

Computer Components (Basic) 1 of 2

View Set

Thyroid, SAIDH, Diabetes, Graves', Cushing's, Addison's (1 of 2) NCLEX

View Set

#03 ¿Cuál es tu color favorito? NTMS Spanish Czora Morgan

View Set

Chapter 8: Production and Operations ManagementAssignment

View Set